Blackboard EMQs Respiratory Flashcards

1
Q

70 yr old retired boiler maker presents with a 5 year history of exertional dyspnoea and a dry cough. The patient is non-smoker. Examination reveals fine crackles heard at the lung bases

What is the most likely diagnosis?
A. Pernicious anaemia
B. Iron deficiency anaemia
C. Pulmonary TB
D. Bronchopneumonia
E. Congestive cardiac failure
F. Chronic lymphatic leukaemia
G. Pulmonary fibrosis
H. COPD
I. PE
J. Acute lymphoblastic leukaemia
K. Sarcoidosis
A

G. Pulmonary fibrosis

Idiopathic pulmonary fibrosis (previously known as Cryptogenic fibrosing alveolitis) progresses over several years and is characterised by pulmonary scar tissue formation and dyspnoea. Patients complain of a non-productive cough and typically reproducible and predictable SOB on exertion. Boiler makers can come into contact with small organic or inorganic dust particles which is thought to be implicated in the cascade of events leading to IPF. While this patient does not smoke, another risk factor is cigarette smoking which significantly increases the risk of IPF. The mean age of diagnosis is 60-70. End expiratory basal crackles are found on examination. These are described as ‘Velcro-like’ in quality. IPF is also associated with clubbing.

How well did you know this?
1
Not at all
2
3
4
5
Perfectly
2
Q

25 yr old HIV positive man has a productive cough for the last 3 months with haemoptysis and night sweats. CXR shows hilar lymphadenopathy.

What is the most likely diagnosis?
A. Pernicious anaemia
B. Iron deficiency anaemia
C. Pulmonary TB
D. Bronchopneumonia
E. Congestive cardiac failure
F. Chronic lymphatic leukaemia
G. Pulmonary fibrosis
H. COPD
I. PE
J. Acute lymphoblastic leukaemia
K. Sarcoidosis
A

C. Pulmonary TB

HIV infection is a key risk factor for pulmonary TB. It is important to have a high level of suspicion when evaluating patients with risk factors who present with suggestive symptoms. Night sweats, fever, malaise, cough, haemoptysis and erythema nodosum are all suggestive. In the first half of the 20th century, tuberculosis accounted for over 90% of cases of erythema nodosum. Other key risk factors for pulmonary TB include exposure to infection and returning from or being born in a high-risk region such as Asia, Africa and Latin America. If TB is suspected, the patient should be placed in isolation and a CXR obtained with 3 sputum samples cultured for AFB being the gold standard of diagnosis. Culture takes several weeks so sputum smears will be done before culture results are known. Interferon-gamma release assays (IGRAs) are now used by some hospitals to rapidly determine a patient’s TB status. All patients who have TB should be tested for HIV within 2 months of diagnosis.

How well did you know this?
1
Not at all
2
3
4
5
Perfectly
3
Q

12 yr old boy has become increasingly tired over the last month. On examination he looks pale, has a large bruise over his right thigh and a firm palpable liver and spleen.

What is the most likely diagnosis?
A. Pernicious anaemia
B. Iron deficiency anaemia
C. Pulmonary TB
D. Bronchopneumonia
E. Congestive cardiac failure
F. Chronic lymphatic leukaemia
G. Pulmonary fibrosis
H. COPD
I. PE
J. Acute lymphoblastic leukaemia
K. Sarcoidosis
A

J. Acute lymphoblastic leukaemia

ALL presents in children with bone marrow involvement and the associated symptoms or in adults with an anterior mediastinal mass. Bone marrow infiltration leads to a pancytopenia leading to anaemia (reduced red blood cells), haemorrhage (reduced platelets) and infections (reduced mature white blood cells). In ALL, bone marrow is replaced by lymphoblasts. There may also be spread to CNS and testes. ALL is associated with Down’s syndrome. In contrast, CLL presents in older adults and is often asymptomatic, discovered by chance when a FBC is ordered. Smear/smudge cells are seen in peripheral blood smear. CLL is associated with a warm-type AIHA and there is peripheral blood lymphocytosis.

How well did you know this?
1
Not at all
2
3
4
5
Perfectly
4
Q

60 yr old publican smokes 20 a day. He has a 10 year history of having a ‘smoker’s morning cough’ when he expectorates clear sputum. This is worse in winter when it turns green and he has to go to his GP for antibiotics. Examination reveals poor air entry over both lung fields and his PEFR is 210 L/min (reduced by 60%)

What is the most likely diagnosis?
A. Pernicious anaemia
B. Iron deficiency anaemia
C. Pulmonary TB
D. Bronchopneumonia
E. Congestive cardiac failure
F. Chronic lymphatic leukaemia
G. Pulmonary fibrosis
H. COPD
I. PE
J. Acute lymphoblastic leukaemia
K. Sarcoidosis
A

H. COPD

Smoking is the most important risk factor, accounting for 90% of COPD. COPD has an insidious onset and usually presents in older people with a history of cough, wheeze and SOB. This patient appears to have infective exacerbations of his COPD every winter. Patients with COPD are at a higher risk of infections and are vaccinated against influenza annually and pneumococcal pneumonia every 5 years. Spirometry is the gold standard for diagnosis, with FEV1/FVC ratio <70% with no evidence of reversibility (unlike asthma) being indicative.

How well did you know this?
1
Not at all
2
3
4
5
Perfectly
5
Q

A 58-year-old man, who smoked 30 cigarettes a day, presents with a 6-week history of cough, malaise, anorexia and weight loss. Past medical history includes hypertension for which he has taken lisinopril and bendrofluazide for 4 years.

What is the most likely cause of the symptoms?
A. ACE inhibitor
B. Foreign body
C. Bronchiectasis
D. Carcinoma of bronchus
E. Asthma 
F. Postnasal drip
G. COPD
H. Tuberculosis
I. Sarcoidosis
J. Oesophageal reflux
A

D. Carcinoma of bronchus

The history of smoking and weight loss point to a bronchial carcinoma. Initial investigation is with a CXR. Diagnosis relies on pathological confirmation from a tissue sample, often obtained from bronchoscopy. First line treatment aims at surgical resection if possible. Small cell lung cancer is treated with chemotherapy and is associated with SIADH and ectopic ACTH. Non-small cell lung cancer is more often associated with clubbing. Squamous cell carcinoma is associated with PTHrp release and is treated with radiotherapy. Adenocarcinomas are usually located peripherally in the lung and are more common in non-smokers although most cases are still associated with smoking. The paraneoplastic syndromes may include Lambert-Eaton myasthenic syndrome.

How well did you know this?
1
Not at all
2
3
4
5
Perfectly
6
Q

A 45-year-old woman who smokes 25 cigarettes a day is reviewed in the diabetic clinic. She has had a dry cough for 2 months. She is on numerous tablets as her diabetes is complicated by microalbuminuria and hypertension. Her GP had given her a course of antibiotics 2 weeks previously.

What is the most likely cause of the symptoms?
A. ACE inhibitor
B. Foreign body
C. Bronchiectasis
D. Carcinoma of bronchus
E. Asthma 
F. Postnasal drip
G. COPD
H. Tuberculosis
I. Sarcoidosis
J. Oesophageal reflux
A

A. ACE inhibitor

A dry cough is a side effect of ACE inhibitors due to the build up of bradykinin which is normally degraded by ACE. ARB such as losartan will be indicated in this case. ARBs are insurmountable antagonists of AT1 receptors for angiotensin II, preventing its renal and vascular effects.

How well did you know this?
1
Not at all
2
3
4
5
Perfectly
7
Q

A 40-year-old Afro-Caribbean woman presents with bilateral parotid swelling, and painful nodules on the front of the shins. She has a dry cough and slight shortness of breath on exertion.

What is the most likely cause of the symptoms?
A. ACE inhibitor
B. Foreign body
C. Bronchiectasis
D. Carcinoma of bronchus
E. Asthma 
F. Postnasal drip
G. COPD
H. Tuberculosis
I. Sarcoidosis
J. Oesophageal reflux
A

I. Sarcoidosis

Sarcoidosis is a chronic multisystem disease with an unknown aetiology. The painful (mauve) nodules are erythema nodosum. Lupus pernio is another typical skin manifestation of sarcoidosis presenting with indurated plaques with discoloration on the face. Parotid enlargement is a classic feature (involvement of exocrine glands). The dry cough and SOB on exertion indicate pulmonary involvement. CXR will typically show bilateral hilar lymphadenopathy and CXR findings are used in the staging of disease. Additionally, serum calcium and ACE levels may be raised. A transbronchial biopsy is essential for diagnosis in most cases and shows the presence of non-caseating granulomas. Black people have a higher lifetime risk of sarcoidosis, as do those of Scandinavian origin. The mainstay of treatment for severe disease involves systemic corticosteroids.

How well did you know this?
1
Not at all
2
3
4
5
Perfectly
8
Q

An 18-year-old man presents with a night-time cough and shortness of breath while playing football. This has got progressively worse over the previous 2 months.

What is the most likely cause of the symptoms?
A. ACE inhibitor
B. Foreign body
C. Bronchiectasis
D. Carcinoma of bronchus
E. Asthma 
F. Postnasal drip
G. COPD
H. Tuberculosis
I. Sarcoidosis
J. Oesophageal reflux
A

E. Asthma

SOB and the cough, which may wake the patient from sleep combined with the patient’s age and progessive course suggest asthma. Examination can show an expiratory wheeze but may be normal and treatment is step-wise based on BTS guidelines. It is worth noting that in severe exacerbations, the chest may be silent. Night symptoms occur in more severe asthma and symptoms can be exacerbated by exercise. Diagnosis is supported by PEFR variation of at least 20% over 3 days in a week over several weeks or an increase of at least 20% to treatment. New guidelines were published in 2018 and can be found on the NICE website.

How well did you know this?
1
Not at all
2
3
4
5
Perfectly
9
Q

A 30-year-old man, a lifelong non-smoker, presents with a history of at least 6 months of purulent sputum. He has had regular chest infections since an attack of measles at the age of 14.

What is the most likely cause of the symptoms?
A. ACE inhibitor
B. Foreign body
C. Bronchiectasis
D. Carcinoma of bronchus
E. Asthma 
F. Postnasal drip
G. COPD
H. Tuberculosis
I. Sarcoidosis
J. Oesophageal reflux
A

C. Bronchiectasis

Bronchiectasis is permanent bronchi dilatation due to bronchial wall damage and loss of elasticity. It is often as a consequence of recurrent/severe infections and most present with chronic productive mucopurulent cough. The most common identifiable cause is CF. Chest CT is the diagnostic test. Diagnosis is aided by sputum analysis. Have a think about what you would expect to hear on ascultation of the chest.

How well did you know this?
1
Not at all
2
3
4
5
Perfectly
10
Q

50 yr old male smoker presents with 3 month hx of cough, haemoptysis and wt loss. Chest examination = unremarkable

What is the most likely diagnosis?
A. Extrinsic allergic alveolitis
B. Chronic bronchitis
C. Fibrosing alveolitis
D. Right ventricular failure
E. Pneumonia
F. Bronchial carcinoma
G. TB
H. ACE inhibitor 
I. Left ventricular failure
J. Influenza
K. Asthma
A

F. Bronchial carcinoma

The history of smoking, weight loss, cough and haemoptysis point to a bronchial carcinoma. Initial investigation is with a CXR. Diagnosis relies on pathological confirmation from a tissue sample, often obtained from bronchoscopy. First line treatment aims at surgical resection if possible. Small cell lung cancer is treated with chemotherapy and is associated with SIADH and ectopic ACTH. Non-small cell lung cancer is more often associated with clubbing. Squamous cell carcinoma is associated with PTHrp release and is treated with radiotherapy. Adenocarcinomas are usually located peripherally in the lung and are more common in non-smokers although most cases are still associated with smoking. The paraneoplastic syndromes may include Lambert-Eaton myasthenic syndrome.

How well did you know this?
1
Not at all
2
3
4
5
Perfectly
11
Q

5yr old coughs most nights. He has frequent courses of antibiotics for a ‘bad chest’ especially in winter. He also has eczema

What is the most likely diagnosis?
A. Extrinsic allergic alveolitis
B. Chronic bronchitis
C. Fibrosing alveolitis
D. Right ventricular failure
E. Pneumonia
F. Bronchial carcinoma
G. TB
H. ACE inhibitor 
I. Left ventricular failure
J. Influenza
K. Asthma
A

K. Asthma

These are infective exacerbations of asthma. The eczema is an atopic disease which is a strong risk factor for the development of asthma. A dry night time cough is commonly seen and suggestive. The hyperexpanded chest suggests persistent asthma. A Harrison’s sulcus, though rare, may also be seen. Treatment will be based on paediatric guidelines which differs from adult asthma. If you are really keen, you can look these up.

How well did you know this?
1
Not at all
2
3
4
5
Perfectly
12
Q

50 yr old male smoker has a productive cough with clear sputum most days, especially winter. He has not lost wt. On examination he has hyperexpanded chest and a few scattered wheezes and crackles

What is the most likely diagnosis?
A. Extrinsic allergic alveolitis
B. Chronic bronchitis
C. Fibrosing alveolitis
D. Right ventricular failure
E. Pneumonia
F. Bronchial carcinoma
G. TB
H. ACE inhibitor 
I. Left ventricular failure
J. Influenza
K. Asthma
A

B. Chronic bronchitis

This patient has COPD, which is a progressive disease characterised by not fully reversible airflow limitation. COPD encompasses both emphysema and chronic bronchitis. Cigarette smoking is the most important risk factor. The hyperexpanded chest implies trapping of air due to incomplete expiration. Wheezes and coarse crackles are commonly seen in exacerbations. The cough is often the first symptom a patient complains of and is usually a morning event which is normally productive. The sputum can change quality with exacerbations/infection. Treatment aims at stopping smoking and vaccinating the patient against influence and pneumococcus with options such as bronchodilators or ICS. LTOT improves survival in those with severe COPD with a low PaO2. Lung function tests are key in diagnosis with an obstructive FEV1/FVC ratio <70% being the finding seen.

How well did you know this?
1
Not at all
2
3
4
5
Perfectly
13
Q

40 yr old Asian male has a 2 month hx of cough, haemoptysis, wt loss and night sweats. He has swollen cervical lymph nodes and his trachea deviated to the left.

What is the most likely diagnosis?
A. Extrinsic allergic alveolitis
B. Chronic bronchitis
C. Fibrosing alveolitis
D. Right ventricular failure
E. Pneumonia
F. Bronchial carcinoma
G. TB
H. ACE inhibitor 
I. Left ventricular failure
J. Influenza
K. Asthma
A

G. TB

It is important to have a high level of suspicion when evaluating patients with risk factors who present with suggestive symptoms. Night sweats, fever, malaise, weight loss, cough, haemoptysis and erythema nodosum are all suggestive. This patient’s tracheal deviation may be due to apical fibrosis or a cavitating lesion. The swollen lymph nodes in this patient’s neck may well represent a scrofula. Other key risk factors for pulmonary TB include exposure to infection and returning from or being born in a high-risk region such as Asia, Africa and Latin America. If TB is suspected, the patient should be placed in isolation and a CXR obtained with 3 sputum samples cultured for AFB being the gold standard of diagnosis. Culture takes several weeks so sputum smears will be done before culture results are known. Interferon-gamma release assays (IGRAs) are now used by some hospitals to rapidly determine a patient’s TB status. All patients who have TB should be tested for HIV within 2 months of diagnosis.

How well did you know this?
1
Not at all
2
3
4
5
Perfectly
14
Q

40 yr old man has a hx of hypertension. His anti-hypertensive meds were recently changed due to ankle swelling. He has now developed a dry cough.

What is the most likely diagnosis?
A. Extrinsic allergic alveolitis
B. Chronic bronchitis
C. Fibrosing alveolitis
D. Right ventricular failure
E. Pneumonia
F. Bronchial carcinoma
G. TB
H. ACE inhibitor 
I. Left ventricular failure
J. Influenza
K. Asthma
A

H. ACE inhibitor

A dry cough is a side effect of ACE inhibitors due to the build up of bradykinin which is normally degraded by ACE. ARB such as losartan will be indicated in this case. ARBs are insurmountable antagonists of AT1 receptors for angiotensin II, preventing its renal and vascular effects.

How well did you know this?
1
Not at all
2
3
4
5
Perfectly
15
Q

A 62 year old man presents with progressive breathlessness over many years. He worked in power stations. He has finger clubbing and his chest xray shows a honeycomb apperance.

What is the most likely diagnosis?
A. Tuberculosis
B. Streptococcal pneumonia
C. Sinusitis
D. Bronchiectasis
E. Cystic fibrosis
F. Asthma
G. Idiopathic pulmonary fibrosis
H. Sarcoidosis
I. Granulomatosis with polyangitis
J. Mycoplasma pneumonia
A

G. Idiopathic pulmonary fibrosis

Idiopathic pulmonary fibrosis (previously known as Cryptogenic fibrosing alveolitis) progresses over several years and is characterised by pulmonary scar tissue formation and dyspnoea. Patients complain of a non-productive cough and typically reproducible and predictable SOB on exertion. Work in power stations can involve contact with small organic or inorganic dust particles which is thought to be implicated in the cascade of events leading to IPF. Another risk factor is cigarette smoking which significantly increases the risk of IPF. The mean age of diagnosis is 60-70. End expiratory basal crackles are found on examination. These are described as ‘Velcro-like’ in quality. IPF is also associated with clubbing. A CXR in most will show reticulonodular shadowing consistent with fibrosis. This can be described as a ‘honeycomb’ pattern.

How well did you know this?
1
Not at all
2
3
4
5
Perfectly
16
Q

A 35 year old lady presents with a rash over her face and raised levels of ACE

What is the most likely diagnosis?
A. Tuberculosis
B. Streptococcal pneumonia
C. Sinusitis
D. Bronchiectasis
E. Cystic fibrosis
F. Asthma
G. Idiopathic pulmonary fibrosis
H. Sarcoidosis
I. Granulomatosis with polyangitis
J. Mycoplasma pneumonia
A

J. Sarcoidosis

Sarcoidosis is a chronic multisystem disease with an unknown aetiology. The lesion described is lupus pernio which is a typical skin manifestation of sarcoidosis presenting with indurated plaques with discoloration on the face. Erythema nodosum is another dermatological manifestation. Additionally, serum calcium and ACE levels may be raised. CXR will typically show bilateral hilar lymphadenopathy and CXR findings are used in the staging of disease. A transbronchial biopsy is essential for diagnosis in most cases and shows the presence of non-caseating granulomas. Black people have a higher lifetime risk of sarcoidosis, as do those of Scandinavian origin. The mainstay of treatment for severe disease involves systemic corticosteroids.

How well did you know this?
1
Not at all
2
3
4
5
Perfectly
17
Q

A 28 year old homeless man presents with tiredness and cough for over 3 months. His CXR showed patchy shadows in the upper zones. His sputum grew positive culture on Lowenstein-jensen medium after 4 weeks

What is the most likely diagnosis?
A. Tuberculosis
B. Streptococcal pneumonia
C. Sinusitis
D. Bronchiectasis
E. Cystic fibrosis
F. Asthma
G. Idiopathic pulmonary fibrosis
H. Sarcoidosis
I. Granulomatosis with polyangitis
J. Mycoplasma pneumonia
A

A. Tuberculosis

This patient should be placed in isolation. His sputum culture results suggest TB which is an AFB growing on Lowenstein-Jensen medium. A sputum culture is the most sensitive and specific test for TB and whilst growth takes a long time, the positive culture is diagnostic of TB. Sputum cultures will be repeated during treatment until 2 consecutive negative cultures. Treatment consists of anti-TB medication. Ethambutol should be stopped if the AFB is sensitive to isoniazid and rifampicin. Pyridoxine should be given with isoniazid to help prevent neuropathy. Pyrazinamide is not recommended in acute gout or pregnancy. Treatment of MDR TB involves additional therapy with agents such as amikacin and ethionamide.

How well did you know this?
1
Not at all
2
3
4
5
Perfectly
18
Q

A 19 year old presents with headache and malaise for 1 week. His chest was clear on auscultation. He subsequently developed a cough and was given erythomycin for 1 week

What is the most likely diagnosis?
A. Tuberculosis
B. Streptococcal pneumonia
C. Sinusitis
D. Bronchiectasis
E. Cystic fibrosis
F. Asthma
G. Idiopathic pulmonary fibrosis
H. Sarcoidosis
I. Granulomatosis with polyangitis
J. Mycoplasma pneumonia
A

L. Mycoplasma pneumonia

The atypical presentation (a week of headache and malaise before the cough) and the prescription of erythromycin, a macrolide antibiotic, point to infection with an atypical pneumonia. However, depending on local prescribing policies, first line therapy for a CAP like pneumococcus may also be with a macrolide. Young people who ‘live together’ are commonly affected. The cough often does not resolve and is dry in nature. Symptoms tend to be prolonged and a low-grade fever is a common finding. Mycoplasma is the only atypical pneumonia on the list although there is no reason why this cannot be Legionella or Chlamydia. All can be treated with macrolides although in EMQs Chlamydia tends to be treated with doxycyline. All 3 atypicals are to some extent sensitive to fluoroquinolones and tetracylines too although these cannot be used in pregnancy. Whether they are first or second line therapy depends on the organism.

How well did you know this?
1
Not at all
2
3
4
5
Perfectly
19
Q

A newborn child presents with small bowel obstruction due to meconium ileus

What is the most likely diagnosis?
A. Tuberculosis
B. Streptococcal pneumonia
C. Sinusitis
D. Bronchiectasis
E. Cystic fibrosis
F. Asthma
G. Idiopathic pulmonary fibrosis
H. Sarcoidosis
I. Granulomatosis with polyangitis
J. Mycoplasma pneumonia
A

F. Cystic fibrosis

CF can present in newborns with a failure to pass meconium (early stools) which can even result in bowel obstruction as in this patient. The bowel may even perforate if the patient is very unlucky resulting in a meconium peritonitis. The most conclusive diagnostic test is the sweat test which is pisitive if sweat chloride is >60mmol/L. Serum IRT from a heel prick blood spot allows screening of newborns. CF is a genetic condition with abnormal salt and water transport due to mutations in the CFTR (an apical anion channel). Heterozygotes generally do not demonstrate disease.

How well did you know this?
1
Not at all
2
3
4
5
Perfectly
20
Q

A 42 year old man has a 3 year history of concurrent progressive SOB and joint pains. His bloods show positive rheumatoid factor.

What is the most likely diagnosis?
A. Tuberculosis
B. Streptococcal pneumonia
C. Sinusitis
D. Bronchiectasis
E. Cystic fibrosis
F. Asthma
G. Idiopathic pulmonary fibrosis
H. Sarcoidosis
I. Granulomatosis with polyangitis
J. Mycoplasma pneumonia
A

I. Granulomatosis with polyangitis

Granulomatosis with polyangitis is a systemic vasculitis affecting the small and medium vessels and presents with the classic triad of upper respiratory tract involvement, lower respiratory tract involvement and GN. It is a multisystem disease which can manifest with symptoms ranging from cutaneous, musculoskeletal, ocular and neurological features. Joint swelling and tenderness may all be present. It is associated with a positive cANCA and rhematoid factor is positive in around half.

If the shortness of breath began some time after the joint pains, instead of occurring concurrently, you might consider lung fibrosis secondary to rheumatoid arthritis

How well did you know this?
1
Not at all
2
3
4
5
Perfectly
21
Q

A 38 year old woman with a history of multiple allergies is given intravenous contrast medium for a urogram. Within a couple of minutes, she has become breathless, with wheeze & stridor, & her blood pressure is 80/40 mmHg.

What is the most likely diagnosis?
A. Pneumothorax
B. Exacerbation of COPD
C. Asthma
D. Acute anxiety
E. Left ventricular failure
F. Anaphylaxis
G. Pulmonary embolus
H. Inhaled foreign body
I. Epiglotitis
J. Viral pneumonia
A

F. Anaphylaxis

There is a sudden onset of both respiratory and cardiovascular complaints with the recently given IV contrast agent. This patient is having an anaphylactic reaction and the airway needs to be promptly secured with prompt treatment with adrenaline. IM adrenaline must not be delayed and the anterolateral thigh is the preferred location, with repeated doses as necessary every 10 minutes or so. A 1:1000 solution is used of 0.3-0.5mg adrenaline. Fluid replacement with IV saline is also indicated to correct the intravascular volume redistribution.

How well did you know this?
1
Not at all
2
3
4
5
Perfectly
22
Q

A 69 year old man with a history of hypertension & stable angina wakes up severely short of breath. He is slightly more comfortable sitting upright. On examination, his blood pressure is 195/115 mmHg & there are crepitations at the bases of both lungs.

What is the most likely diagnosis?
A. Pneumothorax
B. Exacerbation of COPD
C. Asthma
D. Acute anxiety
E. Left ventricular failure
F. Anaphylaxis
G. Pulmonary embolus
H. Inhaled foreign body
I. Epiglotitis
J. Viral pneumonia
A

E. Left ventricular failure

This patient has LVF and pulmonary oedema. This accounts for the basal crepitations heard on ascultation and the SOB which is better on sitting upright. Patients need to be sat upright for this reason and IV access needs to be established. Oxygen, morphone, diuretics (frusemide or another loop diuretic) and nitrates will be given. Once stable, medical treatment of heart failure should be started which involves in the first instance, an ACE inhibitor followed by beta blockade. Ongoing diuretics are necessary if the patient has persistent symptoms of fluid overload.

How well did you know this?
1
Not at all
2
3
4
5
Perfectly
23
Q

A 73 year old woman is recovering in hospital 3 days after an operation to replace her right hip. On going to the toilet she suddenly becomes extremely short of breath & rapidly loses consciousness. Blood pressure is 60/20 mmHg.

What is the most likely diagnosis?
A. Pneumothorax
B. Exacerbation of COPD
C. Asthma
D. Acute anxiety
E. Left ventricular failure
F. Anaphylaxis
G. Pulmonary embolus
H. Inhaled foreign body
I. Epiglotitis
J. Viral pneumonia
A

G. Pulmonary embolus

This patient should be thrombolysed immediately if not contraindicated due to her haemodynamically unstable and critical state. Treatment should not be delayed in this obvious PE. This patient is now at a serious risk of cardiac arrest. Anticoagulation should also be started. The underlying pathophysiology is based on Virchow’s triad. SOB is a common symptom and there may also be chest pain and haemoptysis. This patient has had recent surgery, particularly orthopaedic surgery, which is a strong risk factor for PE. Other strong risk factors include DVT, obesity, prolonged bed rest, malignancy, previous VTE, pregnancy and the thrombophilias such as factor V Leiden. ECG may be normal, or may show tachycardia, new RAD, new RBBB or the classical S wave in I, Q wave with T inversion in III. Various clinical probability scores exist for PE and D-dimer can be used to exclude PE as a diagnosis. The study of choice is a CTPA with direct visualisation of the thrombus. If there is a contraindication to a CT scan such as contrast allergy or pregnancy, then a V/Q scan is indicated. If a V/Q scan is not possible, alternatives such as MRA can be requested. It is worth noting that in patients with cardiopulmonary disease, these tests may not be accurate. A TTE can also be used to detect RV strain seen with PE.

How well did you know this?
1
Not at all
2
3
4
5
Perfectly
24
Q

A 77 year old former coal miner has 30 year history of cough, mostly productive of sputum. He suddenly becomes breathless after a bout of coughing & complains of right sided chest pain. On examination, he is cyanosed, the trachea is deviated to the left & no breath sounds are audible over part of the right side of the chest.

What is the most likely diagnosis?
A. Pneumothorax
B. Exacerbation of COPD
C. Asthma
D. Acute anxiety
E. Left ventricular failure
F. Anaphylaxis
G. Pulmonary embolus
H. Inhaled foreign body
I. Epiglotitis
J. Viral pneumonia
A

A. Pneumothorax

This patient has developed a right sided tension pneumothorax and will need emergency intervention in the form of the insertion of a large bore cannula into the 2nd intercostal space in the MCL of the affected side. This will need to be followed by the insertion of a chest drain.

Otherwise if this was not a tension pneumothorax, for secondary spontaneous pneumothoraces, if large enough for a chest drain or the patient is clinically unstable, chest drain insertion is indicated. Simple aspiration success rate is reduced in secondary spontaneous pneumothoraces. Primary pneumothoraces occur in young people without known lung conditions. This patient has pulmonary fibrosis. Those who suffer recurrent pneumothoraces may have to undergo pleurodesis to stick the parietal and visceral pleural together by an inflammatory reaction.

How well did you know this?
1
Not at all
2
3
4
5
Perfectly
25
Q

A 33 year old airline steward presents with a 1 week history of fever, dry cough, & shortness of breath. On examination, he is tachypnoeic. His lungs are clear to auscultation.

What is the most likely diagnosis?
A. Atypical pneumonia
B. Exacerbation of chronic bronchitis
C. Metastatic carcinoma
D. Acute pulmonary oedema
E. Anaemia
F. Valvular disease
G. Bronchial asthma
H. Bronchial carcnioma
I. Pulmonary embolus
A

A. Atypical pneumonia

This is an atypical pneumonia demonstrated by the history and examination findings. Atypicals include Mycoplasma, Chlamydophila and Legionella. Each one has their own associations, for instance, Mycoplasma is associated with a cold type AIHA. Chlamydophila pneumoniae is associated with otitis, pharyngitis and hoarseness prior to respiratory symptoms and psittaci is associated with birds as vectors. Legionella is associated with hyponatraemia, deranged LFTs and altered consciousness.

How well did you know this?
1
Not at all
2
3
4
5
Perfectly
26
Q

A 65 year old man with a history of chronic productive cough now presents to A&E short of breath & drowsy.

What is the most likely diagnosis?
A. Atypical pneumonia
B. Exacerbation of chronic bronchitis
C. Metastatic carcinoma
D. Acute pulmonary oedema
E. Anaemia
F. Valvular disease
G. Bronchial asthma
H. Bronchial carcinoma
I. Pulmonary embolus
A

B. Exacerbation of chronic bronchitis

This is an exacerbation of COPD. A SABA is indicated as first line with corticosteroids. Antibiotics should be commenced.

How well did you know this?
1
Not at all
2
3
4
5
Perfectly
27
Q

A 60 year old woman presents with dyspnoea. On examination, she has a firm mass in the left breast & decreased breath sounds in the right lower lung fields. Chest x-ray reveals a pleural effusion.

What is the most likely diagnosis?
A. Atypical pneumonia
B. Exacerbation of chronic bronchitis
C. Metastatic carcinoma
D. Acute pulmonary oedema
E. Anaemia
F. Valvular disease
G. Bronchial asthma
H. Bronchial carcinoma
I. Pulmonary embolus
A

C. Metastatic carcinoma

This is pulmonary involvement secondary to a breast primary i.e. metastatic breast cancer. The effusion in this case would be exudative (it is a malignant pleural effusion). Make sure you know the difference between a transudate and an exudate and some causes of each. Metastases are normally round in appearance on CXR. A biopsy can help in identifying the source if unclear.

How well did you know this?
1
Not at all
2
3
4
5
Perfectly
28
Q

A 50 year old male patient on the ward awakes with dyspnoea & frothy sputum. He had suffered an MI a week earlier. On examination, he is cyanosed & tachypnoeic. Auscultation of the lung reveals crepitations.

What is the most likely diagnosis?
A. Atypical pneumonia
B. Exacerbation of chronic bronchitis
C. Metastatic carcinoma
D. Acute pulmonary oedema
E. Anaemia
F. Valvular disease
G. Bronchial asthma
H. Bronchial carcinoma
I. Pulmonary embolus
A

D. Acute pulmonary oedema

This patient has acute pulmonary oedema (the classic finding of pink frothy sputum). This is likely to be caused by LVF secondary to his MI a week earlier. This accounts for the basal crepitations heard on ascultation. Patients need to be sat upright to improve the SOB and IV access needs to be established. Oxygen, morphone, diuretics (frusemide or another loop diuretic) and nitrates will be given. Once stable, medical treatment of heart failure should be started which involves in the first instance, an ACE inhibitor followed by beta blockade. Ongoing diuretics are necessary if the patient has persistent symptoms of fluid overload.

How well did you know this?
1
Not at all
2
3
4
5
Perfectly
29
Q

A 40 year old man presents with cough & breathlessness. Chest x-ray demonstrates diffuse consolidation of the right lower lobe. Despite treatment with iv co-amoxiclav, the fever persists. Chest x-ray shows expansion.

What is the most likely diagnosis?
A. Atypical pneumonia
B. Exacerbation of chronic bronchitis
C. Metastatic carcinoma
D. Acute pulmonary oedema
E. Anaemia
F. Valvular disease
G. Bronchial asthma
H. Bronchial carcinoma
I. Pulmonary embolus
A

A. Atypical pneumonia

Again, this is an atypical pneumonia which is not responding to a common antibiotic given for CAP with broad spectrum action. A macrolide is indicated for atypical pneumonia although the current guidelines do recommend the empiral use of a macrolide for CAP to ensure atypical cover, although local prescribing guidelines do vary.

How well did you know this?
1
Not at all
2
3
4
5
Perfectly
30
Q

A 25 year old man has a 3 day history of shivering, general malaise & productive cough. The x-ray shows right lower lobe consolidation.

What is the most likely cause?
A. Mixed growth of organisms
B. Mycoplasma pneumoniae
C. Streptococcus pneumoniae
D. Mycobacterium tuberculosis
E. Bacteroides fragilis
F. Haemophilus influenzae
G. E coli
H. Staphylococcus aureus
I. Pneumocystis jirovecii
J. Legionella pneumophila
K. Coxiella burnetii
A

C. Streptococcus pneumoniae

Classic lobar pneumonia with no signs and symptoms to suggest an atypical organism is most likely to due to pneumococcus. Symptoms include chills, fever, cough, SOB and pleuritic chest pain. A CXR is the most specific and sensitive test available and antibiotics are indicated.

How well did you know this?
1
Not at all
2
3
4
5
Perfectly
31
Q

A 26 year old man presents with severe shortness of breath and a dry cough which he has had for several weeks. He is an IV drug user. There are purple patches on the arms and in the mouth. CXR shows reticular perihilar opacities. Chest examination is unremarkable.

What is the most likely cause?
A. Mixed growth of organisms
B. Mycoplasma pneumoniae
C. Streptococcus pneumoniae
D. Mycobacterium tuberculosis
E. Bacteroides fragilis
F. Haemophilus influenzae
G. E coli
H. Staphylococcus aureus
I. Pneumocystis jirovecii
J. Legionella pneumophila
K. Coxiella burnetii
A

I. Pneumocystis jirovecii

PCP is caused by Pneumocystis jirovecii, previously called Pneumocystis carinii. It is a fungal organism and an AIDS defining illness. Signs and symptoms occur in a patient who is immunosuppressed especially HIV with a CD4 count <200 with the risk increasing with lower CD4 counts. Currently, those who get PCP are either unaware they have HIV, fail to seek help for their HIV or fail to comply with HAART. PCP does also occur in HIV negative patients (with a more rapid and severe course), but rarely, in conditions such as inflammatory conditions, organ transplant recipients on immunosuppressants or those with blood malignancies. This patient is an IVDU which is a risk factors for HIV. Other risks include unprotected sexual intercourse, especially receptive anal intercourse. The purple patches are KS which is another AIDS defining condition. HIV positive patients tend to present with an insidious onset of symptoms such as non-productive cough and SOB over many weeks. CXR can show opacities in the perihilar region but may be diffuse and the picture is highly variable from a normal CXR to lobar consolidation or nodular lesions. Diagnosis is made on detecting the ‘boat-shaped’ organisms on BAL or induced sputum. Treatment is with co-trimoxazole, which is a 1:5 ratio of trimethoprim/sulphamethoxazole.

How well did you know this?
1
Not at all
2
3
4
5
Perfectly
32
Q

A 35 year old previously healthy man returned from a conference in the USA 5 days ago. He travels frequently and gives a 30 pack year history. He presents with mild confusion, a productive cough, diarrhoea and is pyrexic. His chest examination is normal. CXR shows infiltrates in the RUL.

What is the most likely cause?
A. Mixed growth of organisms
B. Mycoplasma pneumoniae
C. Streptococcus pneumoniae
D. Mycobacterium tuberculosis
E. Bacteroides fragilis
F. Haemophilus influenzae
G. E coli
H. Staphylococcus aureus
I. Pneumocystis jirovecii
J. Legionella pneumophila
K. Coxiella burnetii
A

J. Legionella pneumophila

Legionella is a gram negative rod. Legionella infecting the lungs is legionnaires’ disease or Legionella pneumonia whereas non-lung infection is known as Pontiac fever. This bacteria is found in aqueous environments such as lakes and almost all cases are from contaminated water systems, which relates to the risk factors of getting Legionella (recent water exposure like a hot tub). Smoking is also a risk factor. It can cause confusion as well as hyponatraemia, abdominal pain, diarrhoea and bradycardia. Legionella does not grow on routine culture media and diagnosis relies on urine antigen detection, serology or culture on special media.

How well did you know this?
1
Not at all
2
3
4
5
Perfectly
33
Q

A 20 year old previously healthy woman presents with general malaise, severe cough & breathlessness which has not improved with a 7 day course of amoxicillin. There is nothing significant to find on examination. The x-ray shows patchy shadowing throughout the lung fields. The blood film shows clumping of red cells with suggestion of cold agglutinins.

What is the most likely cause?
A. Mixed growth of organisms
B. Mycoplasma pneumoniae
C. Streptococcus pneumoniae
D. Mycobacterium tuberculosis
E. Bacteroides fragilis
F. Haemophilus influenzae
G. E coli
H. Staphylococcus aureus
I. Pneumocystis jirovecii
J. Legionella pneumophila
K. Coxiella burnetii
A

B. Mycoplasma pneumoniae

The cold agglutinins is what gives this question away. Mycoplasma is associated with with cold type agglutinins and a cold AIHA. Humans are thought to be the only host for Mycoplasma. The most commonly affected are young adults living in close proximity to each other. PCR can be used in diagnosis.

How well did you know this?
1
Not at all
2
3
4
5
Perfectly
34
Q

A 63 year old tramp presents to the A&E department with a 4 day history of haemoptysis. He has felt unwell for about 2 months with a cough, loss of weight & generalised weakness. He attributes his diplopia, which started a fortnight ago, to excessive alcohol consumption. On examination he has bilateral ptosis & proximal weakness in the limbs which improves on repeated testing.

What is the most likely diagnosis?
A. Mesothelioma
B. Streptococcal pneumonia
C. Goodpasture’s disease
D. Pulmonary abscess
E. Microscopic polyarteritis
F. Pulmonary metastases
G. Squamous cell carcinoma
H. Tuberculosis
I. Myaesthenia gravis
J. Pulmonary embolism
K. Small cell carcinoma
A

K. Small cell carcinoma

First line treatment aims at surgical resection if possible. Small cell lung cancer is treated with chemotherapy and is also associated with SIADH and ectopic ACTH. Non-small cell lung cancer is more often associated with clubbing. Squamous cell carcinoma is associated with PTHrp release and is treated with radiotherapy. Adenocarcinomas are usually located peripherally in the lung and are more common in non-smokers although most cases are still associated with smoking. The paraneoplastic syndromes may include Lambert-Eaton myasthenic syndrome which this patient has (though weakness of the eye muscles is uncommon in Lambert-Eaton and is more prominent in myasthenia gravis). This classically presents with weakness which improves on repeated testing (in contrast to myasthenia gravis) and is more commonly associated with small cell lung cancer than other lung cancers. It is for this reason that the most likely diagnosis is small cell and not squamous cell lung cancer.

How well did you know this?
1
Not at all
2
3
4
5
Perfectly
35
Q

A 20 year old man with cystic fibrosis presents to the chest clinic with haemoptysis. He has felt unwell for a fortnight with increased sputum production, fever & rigors. Gram stain of the sputum shows Gram-positive cocci in clusters.

What is the most likely diagnosis?
A. Mesothelioma
B. Streptococcal pneumonia
C. Goodpasture’s disease
D. Pulmonary abscess
E. Microscopic polyarteritis
F. Pulmonary metastases
G. Squamous cell carcinoma
H. Tuberculosis
I. Myaesthenia gravis
J. Pulmonary embolism
K. Small cell carcinoma
A

D. pulmonary abscess

A lung abscess is diagnosed on CXR with a cavitation with an air-fluid level in it. Preceding pneumonia which a patient with CF is at risk of is a risk factor. Of gram positive cocci, staphylococcus occurs in grape-like clusters (this patient) whereas streptococcus occurs in chains. It is worth learning your gram stains for the main organisms. It is worth noting that Staphylococcus aureus is coagulase positive (also Yersinia pestis which causes plague) and Streptoccus pneumoniae is optochin sensitive. Fever and a productive cough are common symptoms and treatment involves antibacterials and drainage/resection if unresponsive. Lung abscesses are commonly caused by aspiration of gastric contents.

How well did you know this?
1
Not at all
2
3
4
5
Perfectly
36
Q

A 48 year old woman with ovarian carcinoma presents to the A&E department with a 12 hour history of haemoptysis associated with dyspnoea & pleuritic pains. On examination she is apyrexial & has a right sided pleural rub. The chest x-ray shows a wedge shaped infarct peripherally on the right but is otherwise normal.

What is the most likely diagnosis?
A. Mesothelioma
B. Streptococcal pneumonia
C. Goodpasture’s disease
D. Pulmonary abscess
E. Microscopic polyarteritis
F. Pulmonary metastases
G. Squamous cell carcinoma
H. Tuberculosis
I. Myaesthenia gravis
J. Pulmonary embolism
K. Small cell carcinoma
A

J. Pulmonary embolism

This CXR finding is Hampton’s hump seen in about a third of PE. Additional CXR signs include Westermark’s sign and Fleischner’s sign. CXR is however not diagnostic and may well be normal. Patients with a high clinical suspicion of PE should be anticoagulated while waiting a definitive diagnosis unless contraindicated. The underlying pathophysiology is based on Virchow’s triad. SOB and chest pain are common symptoms and there may also be haemoptysis. Strong risk factors include DVT, obesity, surgery in the past 2 months, prolonged bed rest, malignancy (which this patient has), previous VTE, pregnancy and the thrombophilias such as factor V Leiden. ECG may be normal, or may show tachycardia, new RAD, new RBBB or the classical S wave in I, Q wave with T inversion in III. Various clinical probability scores exist for PE and D-dimer can be used to exclude PE as a diagnosis. The study of choice is a CTPA with direct visualisation of the thrombus. If there is a contraindication to a CT scan such as contrast allergy or pregnancy, then a V/Q scan is indicated. If a V/Q scan is not possible, alternatives such as MRA can be requested. It is worth noting that in patients with cardiopulmonary disease, these tests may not be accurate. A TTE can also be used to detect RV strain seen with PE.

37
Q

A 51 year old social worker presents to her GP with haemoptysis. On further questioning she admits to having a productive cough for 6 months & to losing 2 stone in weight over the same time. Chest x-ray shows patchy consolidation & scarring in both apices.

What is the most likely diagnosis?
A. Mesothelioma
B. Streptococcal pneumonia
C. Goodpasture’s disease
D. Pulmonary abscess
E. Microscopic polyarteritis
F. Pulmonary metastases
G. Squamous cell carcinoma
H. Tuberculosis
I. Myaesthenia gravis
J. Pulmonary embolism
K. Small cell carcinoma
A

H. Tuberculosis

It is important to have a high level of suspicion when evaluating patients with risk factors who present with suggestive symptoms. Night sweats, fever, malaise, cough, haemoptysis and erythema nodosum are all suggestive. Other key risk factors for pulmonary TB include exposure to infection and returning from or being born in a high-risk region such as Asia, Africa and Latin America. If TB is suspected, the patient should be placed in isolation and a CXR obtained with 3 sputum samples cultured for AFB being the gold standard of diagnosis. Culture takes several weeks so sputum smears will be done before culture results are known. Interferon-gamma release assays (IGRAs) are now used by some hospitals to rapidly determine a patient’s TB status. All patients who have TB should be tested for HIV within 2 months of diagnosis. CXR is the first line test to order. Classically, in primary disease there are middle and lower zone infiltrates. Post-primary TB usually involves apical changes with or without cavitation. However, recent students have indicated that both presentations are seen in both primary and post-primary TB. HIV positive patients tend to have a more atypical CXR including effusion, lower zone involvement and a miliary pattern.

38
Q

A 34 year old man presents to the A&E department with a short history of haemoptysis. He has had a cough for a fortnight & noticed his ankles beginning to swell 5 days ago. Initial blood tests show a creatinine of 400mol/l. An autoantibody screen is positive for p-ANCA & anti-glomerular basement membrane antibodies.

What is the most likely diagnosis?
A. Mesothelioma
B. Streptococcal pneumonia
C. Goodpasture’s disease
D. Pulmonary abscess
E. Microscopic polyarteritis
F. Pulmonary metastases
G. Squamous cell carcinoma
H. Tuberculosis
I. Myaesthenia gravis
J. Pulmonary embolism
K. Small cell carcinoma
A

C. Goodpasture’s disease

Goodpasture’s is associated with anti-GBM antibodies, and of those who are positive, some will have a positive ANCA too, although this is more suggestive of diagnoses such as Churg-Strauss and microscopic polyarteritis. Definitive diagnosis is by renal biopsy showing crescentic GN and linear IgG staining on immunofluorescence. It is one of the few causes of pulmonary renal syndrome. Aggressive treatment is often needed and plasma exchange can also be performed to remove preformed antibodies.

39
Q

Trachea deviated to the right. Hyper-resonant percussion on the left side with reduced breath sounds.

What is the most likely diagnosis?
A. Lobar pneumonia
B. Lobar collapse
C. Normal variant
D. Pleural effusion
E. Pneumothorax
F. Emphysema
G. Chronic bronchitis
H. Pleurisy
I. Pulmonary oedema
J. Idiopathic pulmonary fibrosis
K. Hyperventilation
A

E. Pneumothorax

A tension pneumothorax shifts the trachea AWAY with hyper-resonance on affected side and reduced/absent breath sounds. A tracheal shift means this is a tension pneumothorax which needs immediate relief with a wide bore cannula inserted into the second intercostal space in the MCL.

40
Q

Trachea deviated to left. Dull to percussion & reduced breath sounds at left base.

What is the most likely diagnosis?
A. Lobar pneumonia
B. Lobar collapse
C. Normal variant
D. Pleural effusion
E. Pneumothorax
F. Emphysema
G. Chronic bronchitis
H. Pleurisy
I. Pulmonary oedema
J. Idiopathic pulmonary fibrosis
K. Hyperventilation
A

B. Lobar collapse

Collapse pulls the trachea TOWARDS the affected side. There is dullness and reduced/absent breath sounds due to a lack of air filled lung in this space. Do you know how to identify which lobe has collapsed on a CXR? The findings in this examination are consistent with LLL collapse. A ‘sail sign’ will classically be seen behind the cardiac shadow on CXR.

41
Q

Reduced chest movements bilaterally. Using accessory muscles of respiration. Breath sounds generally quiet. Hyper-expanded lung fields seen on CXR

What is the most likely diagnosis?
A. Lobar pneumonia
B. Lobar collapse
C. Normal variant
D. Pleural effusion
E. Pneumothorax
F. Emphysema
G. Chronic bronchitis
H. Pleurisy
I. Pulmonary oedema
J. Idiopathic pulmonary fibrosis
K. Hyperventilation
A

G. Chronic bronchitis

This patient is in obvious respiratory distress and is using accessory muscles of respiration. Chest movements are reduced too. There is hyperexpansion of the lung fields due to emphysema, which has caused the respiratory distress, combined with diminished breath sounds.

42
Q

Bilateral fine basal crepitations. There are no signs of CCF.

What is the most likely diagnosis?
A. Lobar pneumonia
B. Lobar collapse
C. Normal variant
D. Pleural effusion
E. Pneumothorax
F. Emphysema
G. Chronic bronchitis
H. Pleurisy
I. Pulmonary oedema
J. Idiopathic pulmonary fibrosis
K. Hyperventilation
A

J. Idiopathic pulmonary fibrosis

Bibasal inspiratory crackles without signs of CCF are seen in idiopathic pulmonary fibrosis. These are described as ‘Velcro-like’ in quality. IPF is also associated with clubbing. Idiopathic pulmonary fibrosis (previously known as Cryptogenic fibrosing alveolitis) progresses over several years and is characterised by pulmonary scar tissue formation and dyspnoea. Patients complain of a non-productive cough and typically reproducible and predictable SOB on exertion

43
Q

Trachea central. Reduced chest movement on right. Dull to percussion on right. Bronchial breathing at right base.

What is the most likely diagnosis?
A. Lobar pneumonia
B. Lobar collapse
C. Normal variant
D. Pleural effusion
E. Pneumothorax
F. Emphysema
G. Chronic bronchitis
H. Pleurisy
I. Pulmonary oedema
J. Idiopathic pulmonary fibrosis
K. Hyperventilation
A

A. Lobar pneumonia

These are the classic signs of pneumonia. Also expect to find increased vocal resonance and tactile vocal fremitus over areas of consolidation. In reality, it can be confusing if the pneumonia causes a lobar collapse as you can also find signs of collapse on examination. Always consider the history as well as examination and investigation findings. On a CXR with pneumonia, you can expect to see air space shadowing with air bronchograms. Always remember to auscultate at the right axilla when doing a respiratory examination or a RML pneumonia may be missed.

44
Q

A 50 year old man became suddenly breathless whilst eating. He has marked stridor & is choking & drooling.

What is the most appropriate management?
A. Nebulised salbutamol
B. Low molecular weight heparin
C. Intravenous insulin
D. Intravenous aminophylline
E. Pleural aspiration
F. Rapid infusion of saline
G. Intravenous adrenaline
H. Heimlich manoeuvre
I. Intravenous furosemide
J. Chest drain
K. Re-breathing into paper bag
L. Forced alkaline diuresis
A

H. Heimlich manoeuvre

This patient has choked on some food. The patient should be encouraged to cough if they are conscious. Otherwise, external manoevres can be performed such as abdominal thrusts (Heimlich) or back blows. These actions increase intrathoracic pressure and help to dislodge the foreign body. If it still isn’t removed, a flexible bronchoscopy may be necessary. Most cases occur in very young children.

45
Q

A 60 year old male presents with acute breathlessness & a cough productive of frothy, pink sputum. He cannot lie flat. On examination, he has crackles to both midzones & a few scattered wheezes.

What is the most appropriate management?
A. Nebulised salbutamol
B. Low molecular weight heparin
C. Intravenous insulin
D. Intravenous aminophylline
E. Pleural aspiration
F. Rapid infusion of saline
G. Intravenous adrenaline
H. Heimlich manoeuvre
I. Intravenous furosemide
J. Chest drain
K. Re-breathing into paper bag
L. Forced alkaline diuresis
A

I. Intravenous furosemide

This patient has pulmonary oedema. CXR may show pulmonary vascular redistribution to the upper zones, Kerley B lines, an increased CTR (cardiomegaly) and pleural effusion.

46
Q

A 20 year old woman is too breathless to speak. Her pulse is 120/min, respiratory rate 30 per min & peak expiratory flow is 100l/min. Examination reveals a very quiet chest & chest x-ray is normal.

What is the most appropriate management?
A. Nebulised salbutamol
B. Low molecular weight heparin
C. Intravenous insulin
D. Intravenous aminophylline
E. Pleural aspiration
F. Rapid infusion of saline
G. Intravenous adrenaline
H. Heimlich manoeuvre
I. Intravenous furosemide
J. Chest drain
K. Re-breathing into paper bag
L. Forced alkaline diuresis
A

A. Nebulised salbutamol

This patient is having an asthma attack. This patient is too breathless to speak and has a quiet chest so this is severe and ICU admission is indicated. Initial treatment is with repeated administration of an inhaled SABA with early systemic corticosteroids and supplemental oxygen, monitoring the patient’s status regularly.

47
Q

A 25 year old woman has just returned from a holiday in Kenya. She suddenly became breathless & is cyanosed. Her pulse is 120/min, BP 110/70, peak expiratory flow 400l/min. Chest x-ray is normal.

What is the most appropriate management?
A. Nebulised salbutamol
B. Low molecular weight heparin
C. Intravenous insulin
D. Intravenous aminophylline
E. Pleural aspiration
F. Rapid infusion of saline
G. Intravenous adrenaline
H. Heimlich manoeuvre
I. Intravenous furosemide
J. Chest drain
K. Re-breathing into paper bag
L. Forced alkaline diuresis
A

B. Low molecular weight heparin

Patients with a high clinical suspicion of PE should be anticoagulated while waiting a definitive diagnosis unless contraindicated. Fondaparinaux may also be used.

48
Q

A 50 year old with pulmonary fibrosis develops sudden left-sided pleuritic pain & dyspnoea. He has reduced air entry in the left side of the chest & percussion is hyper-resonant. Oxygen saturation is 80%.

What is the most appropriate management?
A. Nebulised salbutamol
B. Low molecular weight heparin
C. Intravenous insulin
D. Intravenous aminophylline
E. Pleural aspiration
F. Rapid infusion of saline
G. Intravenous adrenaline
H. Heimlich manoeuvre
I. Intravenous furosemide
J. Chest drain
K. Re-breathing into paper bag
L. Forced alkaline diuresis
A

J. Chest drain

This patient has developed a pneumothorax. This is a secondary spontaneous pneumothorax, and if large enough for a chest tube or the patient is clinically unstable, chest drain insertion is indicated. Simple aspiration success rate is reduced in secondary spontaneous pneumothoraces. Primary pneumothoraces occur in young people without known lung conditions. This patient has pulmonary fibrosis. Those who suffer recurrent pneumothoraces may have to undergo pleurodesis using a chemical such as doxycycline to stick the perietal and visceral pleural together by an inflammatory reaction.

49
Q

A 36 year old popstar presents with fever, a cough & an itchy vesicular rash. Chest x-ray shows mottling through both lung fields

What is the most likely causative organism?
A. Corynebacterium diphtheriae
B. Haemophilus influenza
C. Group A streptococci
D. Influenzae A
E. Aspergillus fumigatus
F. Legionella pneumophila
G. Mycoplasma pneumoniae
H. Chlamydia pneumoniae
I. Staphylococcus aureus
J. Streptococcus pneumoniae
K. Escherichia coli
L. Pneumocystis jirovecii
M. Adenovirus
N. Varicella zoster
A

N. Varicella zoster

The pruritic vesicular rash (the classic description of a ‘dewdrop on a rose petal’) makes you think of VZV. The rash typically occurs on the patient’s torso and face and pneumonia is a complication occuring more commonly in those with immunosuppression. The lesions are often crusted over by 7-10 days. The diagnosis is based on clinical findings.

50
Q

A 60 year old woman presented to the casualty with chest pain on coughing, fever, shortness of breath for the last 2 days. Her chest x-ray showed right lower lobe consolidation & pleural effusion

What is the most likely causative organism?
A. Corynebacterium diphtheriae
B. Haemophilus influenza
C. Group A streptococci
D. Influenzae A
E. Aspergillus fumigatus
F. Legionella pneumophila
G. Mycoplasma pneumoniae
H. Chlamydia pneumoniae
I. Staphylococcus aureus
J. Streptococcus pneumoniae
K. Escherichia coli
L. Pneumocystis jirovecii
M. Adenovirus
N. Varicella zoster
A

J. Streptococcus pneumonia

This is classic CAP. Classic lobar pneumonia with no signs and symptoms to suggest an atypical organism is most likely to due to pneumococcus. Symptoms include chills, fever, cough, SOB and pleuritic chest pain. A CXR is the most specific and sensitive test available and antibiotics are indicated.

51
Q

30 year old intravenous drug abuser presented to the GP with shortness of breath. The GP sent him for a blood count, HIV testing & a chest x-ray. He was found to be HIV positive. Chest x-ray showed bilateral fluffy opacities.

What is the most likely causative organism?
A. Corynebacterium diphtheriae
B. Haemophilus influenza
C. Group A streptococci
D. Influenzae A
E. Aspergillus fumigatus
F. Legionella pneumophila
G. Mycoplasma pneumoniae
H. Chlamydia pneumoniae
I. Staphylococcus aureus
J. Streptococcus pneumoniae
K. Escherichia coli
L. Pneumocystis jirovecii
M. Adenovirus
N. Varicella zoster
A

L. Pneumocystis jirovecii

PCP is caused by Pneumocystis jirovecii, previously called Pneumocystis carinii. It is a fungal organism and an AIDS defining illness. Signs and symptoms occur in a patient who is immunosuppressed, especially HIV with a CD4 count <200 with the risk increasing with lower CD4 counts. Currently, those who get PCP are either unaware they have HIV, fail to seek help for their HIV or fail to comply with HAART. PCP does also occur in HIV negative patients (with a more rapid and severe course), but rarely, in conditions such as inflammatory conditions, organ transplant recipients on immunosuppressants or those with blood malignancies. This patient is an IVDU which is a risk factors for HIV. Other risks include unprotected sexual intercourse, especially receptive anal intercourse. HIV positive patients tend to present with an insidious onset of symptoms such as non-productive cough and SOB over many weeks. CXR can show opacities in the perihilar region but may be diffuse and the picture is highly variable from a normal CXR to lobar consolidation or nodular lesions. Diagnosis is made on detecting the ‘boat-shaped’ organisms on BAL or induced sputum. Treatment is with co-trimoxazole, which is a 1:5 ratio of trimethoprim/sulphamethoxazole.

52
Q

A plumber renovating old properties presented to casualty with fever & loss of consciousness. On examination he had bilateral consolidation. Plasma sodium was low. The doctor sent for urinary antigen & serology. On the results he was treated with azithromycin & ciprofloxacin & improved.

What is the most likely causative organism?
A. Corynebacterium diphtheriae
B. Haemophilus influenza
C. Group A streptococci
D. Influenzae A
E. Aspergillus fumigatus
F. Legionella pneumophila
G. Mycoplasma pneumoniae
H. Chlamydia pneumoniae
I. Staphylococcus aureus
J. Streptococcus pneumoniae
K. Escherichia coli
L. Pneumocystis jirovecii
M. Adenovirus
N. Varicella zoster
A

F. Legionella pneumophila

Legionella is a gram negative rod. Legionella infecting the lungs is legionnaires’ disease or Legionella pneumonia whereas non-lung infection is known as Pontiac fever. This bacteria is found in aqueous environments such as lakes and almost all cases are from contaminated water systems, which relates to the risk factors of getting Legionella (recent water exposure like a hot tub). Smoking is also a risk factor. It can cause confusion as well as hyponatraemia, abdominal pain, diarrhoea and bradycardia. Legionella does not grow on routine culture media and diagnosis relies on urine antigen detection, serology or culture on special media. Treatment is with fluoroquinolones or macrolides. Azithromycin is considered a safer option than erythromycin. For severe cases, a combination of both is given although this is potentially toxic and can cause long QT and torsades de pointes.

53
Q

A holiday worker had a severe chest infection abroad & was diagnosed to have influenza A infection. He was improving but suddenly deteriorated with the last 24 hours becoming breathless, febrile & septic. X-ray chest showed circular opacities some with a fluid level. Gram stain of sputum showed Gram positive cocci in clusters.

What is the most likely causative organism?
A. Corynebacterium diphtheriae
B. Haemophilus influenza
C. Group A streptococci
D. Influenzae A
E. Aspergillus fumigatus
F. Legionella pneumophila
G. Mycoplasma pneumoniae
H. Chlamydia pneumoniae
I. Staphylococcus aureus
J. Streptococcus pneumoniae
K. Escherichia coli
L. Pneumocystis jirovecii
M. Adenovirus
N. Varicella zoster
A

I. Staphylococcus aureus

Think Staphylococcus aureus for post-influenza pneumonia. It causes a cavitating pneumonia which explains the CXR findings (some abscesses are also seen) and Gram stain of culture yields grape like clusters of Gram positive cocci which is consistent with staphylococcus. Treatment of staphyloccocal infection is with flucoxacillin or vancomycin if MRSA.

54
Q

A 30 year old company executive became unwell whilst on a business trip. He developed a high fever, muscle pains, nausea & vomiting, abdominal pain. He admitted to the hotel doctor that his cough has worsened over the past 7 days & he had coughed up blood on a couple of occasions. Chest x-ray showed consolidation in both lungs.

A. Pneumothorax
B. COPD
C. Atypical pneumonia
D. Sarcoidosis
E. Bronchial asthma
F. Cystic fibrosis
G. Lung abscess
H. Bronchial carcinoma
I. Pleural effusion
J. Fibrosing alveolitis
K. Bronchiectasis
A

C. Atypical pneumonia

Given this is the only pneumonia option on the list, this is an easy question, although the presentation is in line with an atypical organism. The most common atypical pneumonias are Mycoplasma, Legionella and Chlamydophila (Chlamydia). All can be treated with macrolides and are to some extent sensitive to fluoroquinolones and tetracyclines too, although these are contraindicated in pregnancy. Check local prescribing policies first.

55
Q

A 40 year old alcoholic man who was treated for a chest infection one week previously, developed a productive cough, & a fever. He felt unwell. He lost weight during this period of illness & also coughed up blood several times. A chest x-ray showed a fluid level in the right lung.

A. Pneumothorax
B. COPD
C. Atypical pneumonia
D. Sarcoidosis
E. Bronchial asthma
F. Cystic fibrosis
G. Lung abscess
H. Bronchial carcinoma
I. Pleural effusion
J. Fibrosing alveolitis
K. Bronchiectasis
A

G. Lung abscess

A lung abscess is diagnosed on CXR with a cavitation with an air-fluid level in it. Preceding pneumonia which this patient gives a history of is a risk factor. Fever and a productive cough are common symptoms and treatment involves antibacterials and drainage/resection if unresponsive. Lung abscesses are commonly caused by aspiration of gastric contents.

56
Q

A 20 year old man with recurrent episodes of chest infection & diarrhoea, which is difficult to flush away in the toilet. He developed a persistent cough with the production of sputum & blood. On examination his fingers are clubbed & in his chest there are low pitched inspiratory & expiratory crackles, plus some wheeze. He recalls being small for his age despite having a healthy appetite.

A. Pneumothorax
B. COPD
C. Atypical pneumonia
D. Sarcoidosis
E. Bronchial asthma
F. Cystic fibrosis
G. Lung abscess
H. Bronchial carcinoma
I. Pleural effusion
J. Fibrosing alveolitis
K. Bronchiectasis
A

F. Cystic fibrosis

CF is autosomal recessive and the mean age of death is around 40. There is currently no cure for this condition. The reccurent chest infections and greasy stools (fat malabsorption due to pancreatic insufficiency) should make you think of CF. A persistent cough which is productive should also raise suspicions. Examination findings here which raise your suspicion include clubbing and crackles on auscultation. Additionally, you may find nasal polyps and hepatomegaly and/or splenomegaly and a congenital absence of the vas deferens in males. There is also some failure to thrive with the patient being small for his age. The most conclusive diagnostic test is the sweat test which is positive if sweat chloride is >60mmol/L. Serum IRT from a heel prick blood spot allows screening of newborns. CF is a genetic condition with abnormal salt and water transport due to mutations in the CFTR (an apical anion channel). Heterozygotes generally do not demonstrate disease.

57
Q

Mr Jones is a 60 year old man who smoked heavily over the past 40 years (up to 40 cigarettes per day). He gives a 4 week history of a cough with the production of sputum & blood, breathlessness chest pain & weight loss. On examination his fingers were clubbed & he had a ptosis in the left eye.

A. Pneumothorax
B. COPD
C. Atypical pneumonia
D. Sarcoidosis
E. Bronchial asthma
F. Cystic fibrosis
G. Lung abscess
H. Bronchial carcinoma
I. Pleural effusion
J. Fibrosing alveolitis
K. Bronchiectasis
A

H. Bronchial carcinoma

This patient is a heavy smoker with a history of respiratory signs and weight loss. The presentation is alluding to a pancoast tumour (most are non-small cell) in the left apex causing ipsilateral Horner’s syndrome (miosis, anhidrosis, ptosis and enophthalmos). A pancoast tumour can also affect the brachial plexus leading to wasting of the intrinsic muscles of the hand as well as a hoarse bovine cough due to compression of the recurrent laryngeal nerve. Initial investigation of bronchial carcinoma is with a CXR. Diagnosis relies on pathological confirmation from a tissue sample, often obtained from bronchoscopy. First line treatment aims at surgical resection if possible. Small cell lung cancer is treated with chemotherapy and is associated with SIADH and ectopic ACTH. Non-small cell lung cancer is more often associated with clubbing. Squamous cell carcinoma is associated with PTHrp release and is treated with radiotherapy. Adenocarcinomas are usually located peripherally in the lung and are more common in non-smokers although most cases are still associated with smoking. The paraneoplastic syndromes may include Lambert-Eaton myasthenic syndrome.

58
Q

John is a 35 year old tennis player who complains of breathlessness after games. His sleep has been disturbed recently by coughing during the night. On examination he was a tall, thin but athletic looking young man. There were high-pitched expiratory wheeze in both lungs.

A. Pneumothorax
B. COPD
C. Atypical pneumonia
D. Sarcoidosis
E. Bronchial asthma
F. Cystic fibrosis
G. Lung abscess
H. Bronchial carcinoma
I. Pleural effusion
J. Fibrosing alveolitis
K. Bronchiectasis
A

E. Bronchial asthma

59
Q

Mark is a 20 year old student who developed acute chest pain with breathlessness after working out at the gym. On examination he was breathless at rest. Expansion of the chest was restricted on one side with hyper-resonant percussion note.

A. Pneumothorax
B. COPD
C. Atypical pneumonia
D. Sarcoidosis
E. Bronchial asthma
F. Cystic fibrosis
G. Lung abscess
H. Bronchial carcinoma
I. Pleural effusion
J. Fibrosing alveolitis
K. Bronchiectasis
A

A. Pneumothorax

This patient has developed a primary pneumothorax. Primary pneumothoraces occur in young people without known lung conditions. The main investigation is a CXR and pneumothoraces are classified by the BTS as large (>2cm visible rim between the lung margin and the chest wall) or small (<2cm). If the patient is clinically stable, they can be observed and given oxygen – an invasive approach is not necessary and the oxygen will increase the rate of pneumothorax reabsorption. A pneumothorax, if left, is something that resolves by itself over time by reabsorption. Cavemen (and cavewomen) who got pneumothoraces all those years ago didn’t just walk around breathless until they died. If large however, percutaneous needle aspiration is required (IV cannula 2nd intercostal space, or 3rd, at the MCL). A CXR should be obtained after this procedure. If this fails, a chest drain should be inserted.

60
Q

A 45 year old doctor from Ethiopia with a 6 week history of fever, drenching night sweats and a cough. He is a heavy smoker. On examination he is thin and looks unwell. He has nicotine stained fingers. Dull to percusion at the right upper zone with reduced breath sounds.

What is the most likely diagnosis?
A. Pulmonary oedema
B. Pleural mesothelioma
C. Pneumothorax
D. Pneumonia
E. Pulmonary tuberculosis
F. Pulmonary embolus
G. Rheumatoid arthritis
H. Acute asthma
I. COPD
J. Aspiration pneumonia
K. Chest injury with rib fractures
L. Sarcoidosis
M. Carcinoma of Bronchus
N. Lung metastases
A

E. Pulmonary tuberculosis

This is pulmonary TB. The patient is from an endemic area and has presented with fever, drenching night sweats and a cough. There is also anorexia and examination findings are consistent with post-primary TB with apical consolidation. CXR is the first line test to order. Classically, in primary disease there are middle and lower zone infiltrates. Post-primary TB usually involves apical changes with or without cavitation. However, recent students have indicated that both presentations are seen in both primary and post-primary TB. HIV positive patients tend to have a more atypical CXR including effusion, lower zone involvement and a miliary pattern.

61
Q

25 year old female with acute onset of chills, fever, cough with brown phlegm for three days. On examination she appears toxic, temperature 40 degrees Celsius, reduced breath sounds, bronchial breathing and stony dullness left lung base.

What is the most likely diagnosis?
A. Pulmonary oedema
B. Pleural mesothelioma
C. Pneumothorax
D. Pneumonia
E. Pulmonary tuberculosis
F. Pulmonary embolus
G. Rheumatoid arthritis
H. Acute asthma
I. COPD
J. Aspiration pneumonia
K. Chest injury with rib fractures
L. Sarcoidosis
M. Carcinoma of Bronchus
N. Lung metastases
A

D. Pneumonia

The rusty coloured phlegm is hinting at a pneumococcal pneumonia.The patient has presented with common symptoms of fever, chills and a cough. There may also be SOB, rigors and pleuritic chest pain. The most specific and sensitive test is a CXR (PA and lateral) and initial treatment of a CAP is empirical with antibiotics. Often diagnosis is made solely on history and examination findings. Bronchial breathing, reduced breath sounds and the presence of a left sided parapneumonic effusion all indicate a pneumonia. Management is guided by the patient’s CURB-65 score.

62
Q

70 year old male ex-builder with progressive pain in his right chest, and with cough and SOB for a few months. CXR shows pleural thickening and right pleural effusion.

What is the most likely diagnosis?
A. Pulmonary oedema
B. Pleural mesothelioma
C. Pneumothorax
D. Pneumonia
E. Pulmonary tuberculosis
F. Pulmonary embolus
G. Rheumatoid arthritis
H. Acute asthma
I. COPD
J. Aspiration pneumonia
K. Chest injury with rib fractures
L. Sarcoidosis
M. Carcinoma of Bronchus
N. Lung metastases
A

B. Pleural mesothelioma

An ex-builder is likely to have a history of asbestos exposure which is the principal risk factor for developing a malignant pleural mesothelioma. CXR shows the presence of a unilateral effusion and irregular pleural thickening. Most patients present with SOB and chest pain. There is a 20-40 year latency period between exposure to asbestos and development of malignancy. Hence, the typical patient presents between 60-80. A more invasive investigation is needed for a definitive diagnosis.

63
Q

A 70 year old female, heavy smoker, for several years who presents with weight loss, reduced appetite and haemoptysis for 1 month. On examination she is thin, afebrile and is clubbed. She has bronchial breathing right upper zone. Reduced breath sounds and dullness on right base. CXR shows right lung collapse with effusion

What is the most likely diagnosis?
A. Pulmonary oedema
B. Pleural mesothelioma
C. Pneumothorax
D. Pneumonia
E. Pulmonary tuberculosis
F. Pulmonary embolus
G. Rheumatoid arthritis
H. Acute asthma
I. COPD
J. Aspiration pneumonia
K. Chest injury with rib fractures
L. Sarcoidosis
M. Carcinoma of Bronchus
N. Lung metastases
A

M. Carcinoma of Bronchus

The history of smoking and weight loss combined with examination findings point to a bronchial carcinoma. First line treatment aims at surgical resection if possible. Small cell lung cancer is treated with chemotherapy and is associated with SIADH and ectopic ACTH. Non-small cell lung cancer is more often associated with clubbing. Squamous cell carcinoma is associated with PTHrp release and is treated with radiotherapy. Adenocarcinomas are usually located peripherally in the lung and are more common in non-smokers although most cases are still associated with smoking. The paraneoplastic syndromes may include Lambert-Eaton myasthenic syndrome.

64
Q

A 50 year old Asian diabetic woman is admitted with increasing shortness of breath and ankle swelling. ECG shows inverted T waves in levels I, AVL and V4-6. Upper lobe blood diversion and bilateral pleural effusions are found on chest X-ray

What is the most likely diagnosis?
A. Pulmonary oedema
B. Pleural mesothelioma
C. Pneumothorax
D. Pneumonia
E. Pulmonary tuberculosis
F. Pulmonary embolus
G. Rheumatoid arthritis
H. Acute asthma
I. COPD
J. Aspiration pneumonia
K. Chest injury with rib fractures
L. Sarcoidosis
M. Carcinoma of Bronchus
N. Lung metastases
A

A. Pulmonary oedema

CXR findings here are consistent with pulmonary oedema. Pulmonary vascular redistribution to the upper zones, Kerley B lines, an increased CTR (cardiomegaly) and pleural effusion may be seen. The patient is also in CCF with evidence of LV dysfunction (SOB) and RV dysfunction (ankle swelling).

65
Q

A 10 year old boy presents to A&E one winter following a school P.E. lesson with a difficulty in breathing and an audible wheeze.

What is the most likely diagnosis?
A. Pulmonary embolism
B. Pulmonary fibrosis
C. Sarcoidosis
D. Emphysema
E. Pneumothorax
F. TB
G. Bronchitis
H. Asthma
I. Lung Tumour
J. Pneumonia
K. Pleural effusion
A

H. Asthma

This is likely to be asthma, exacerbated by the cold air and exercise. Treatment is in this case based on paediatric guidelines which differs from that of adult asthma. Diagnosis is supported by PEFR variation of at least 20% over 3 days in a week over several weeks or an increase of at least 20% to treatment.

66
Q

A 24 year old accountant presented to his GP with a 2 week history of tiredness and a persistant cough and complained of “not being able to complete his normal gym routine”. On examination he was pyrexic, had decreased lung expansion and increased vocal resonance and auscultation revealed bronchial breathing.

What is the most likely diagnosis?
A. Pulmonary embolism
B. Pulmonary fibrosis
C. Sarcoidosis
D. Emphysema
E. Pneumothorax
F. TB
G. Bronchitis
H. Asthma
I. Lung Tumour
J. Pneumonia
K. Pleural effusion
A

J. Pneumonia

This patient has pneumonia. Symptoms include chills, fever, cough, SOB and pleuritic chest pain. Examination findings are consistent with his diagnosis. A CXR is the most specific and sensitive test available and antibiotics are indicated.

67
Q

A 73 year old lady, presented to the respiratory team after a 2 week stay in hospital following a stroke. She was breathless at rest with a central chest pain and was coughing up blood.

What is the most likely diagnosis?
A. Pulmonary embolism
B. Pulmonary fibrosis
C. Sarcoidosis
D. Emphysema
E. Pneumothorax
F. TB
G. Bronchitis
H. Asthma
I. Lung Tumour
J. Pneumonia
K. Pleural effusion
A

A. Pulmonary embolism

Patients with a high clinical suspicion of PE should be anticoagulated while waiting a definitive diagnosis unless contraindicated. The underlying pathophysiology is based on Virchow’s triad. SOB and chest pain are common symptoms and there may also be haemoptysis. Strong risk factors include DVT, obesity, surgery in the past 2 months, prolonged bed rest, malignancy, previous VTE, pregnancy and the thrombophilias such as factor V Leiden. ECG may be normal, or may show tachycardia, new RAD, new RBBB or the classical S wave in I, Q wave with T inversion in III. Various clinical probability scores exist for PE and D-dimer can be used to exclude PE as a diagnosis. The study of choice is a CTPA with direct visualisation of the thrombus. If there is a contraindication to a CT scan such as contrast allergy or pregnancy, then a V/Q scan is indicated. If a V/Q scan is not possible, alternatives such as MRA can be requested. It is worth noting that in patients with cardiopulmonary disease, these tests may not be accurate. A TTE can also be used to detect RV strain seen with PE.

68
Q

A 55 year old male smoker presents to her GP with a 6 month history of a persistent dry cough and unexplained weight loss. On examination she was jaundiced with an enlarged liver.

What is the most likely diagnosis?
A. Pulmonary embolism
B. Pulmonary fibrosis
C. Sarcoidosis
D. Emphysema
E. Pneumothorax
F. TB
G. Bronchitis
H. Asthma
I. Lung Tumour
J. Pneumonia
K. Pleural effusion
A

I. Lung Tumour

This is a smoker with respiratory complaints and unexplained weight loss pointing to malignancy. The hepatomegaly is likely to be nodular due to hepatic secondaries from a primary lung cancer and this hepatic dysfunction has caused this patient’s jaundice. First line treatment aims at surgical resection if possible. Small cell lung cancer is treated with chemotherapy and is associated with SIADH and ectopic ACTH. Non-small cell lung cancer is more often associated with clubbing. Squamous cell carcinoma is associated with PTHrp release and is treated with radiotherapy. Adenocarcinomas are usually located peripherally in the lung and are more common in non-smokers although most cases are still associated with smoking.

69
Q

A Nigerian 26 year old female presents to her doctor with a dry cough and a painful and strange red left shin. A subsequent chest X-ray reveals bilateral hilar lymphadenopathy.

What is the most likely diagnosis?
A. Pulmonary embolism
B. Pulmonary fibrosis
C. Sarcoidosis
D. Emphysema
E. Pneumothorax
F. TB
G. Bronchitis
H. Asthma
I. Lung Tumour
J. Pneumonia
K. Pleural effusion
A

C. Sarcoidosis

Sarcoidosis is a chronic multisystem disease with an unknown aetiology. Lung involvement is very common. The strange red left shin is erythema nodosum and are tender erythematous nodules. Lupus pernio is another typical skin manifestation of sarcoidosis presenting with indurated plaques with discoloration on the face. CXR will typically show bilateral hilar lymphadenopathy and CXR findings are used in the staging of disease. Additionally, serum calcium and ACE levels may be raised. A transbronchial biopsy is essential for diagnosis in most cases and shows the presence of non-caseating granulomas. Black people have a higher lifetime risk of sarcoidosis, as do those of Scandinavian origin. The mainstay of treatment for severe disease involves systemic corticosteroids.

70
Q

A 48 year old male pharmaceutical worker presents with difficulty sleeping. On further questioning you find out that it is due to coughing all night. He is also suffering chest tightness, breathlessness and you can detect a wheeze when he talks to you. Although his wife notes that he is not as bad on the weekend.

What is the most likely diagnosis?
A. Emphysema
B. Tuberculosis
C. HIV
D. Asthma
E. Lung abcess
F. Pancreatitis
G. Lung cancer
H. Bronchiectasis
I. Chronic bronchitis
J. Pneumonia
K. Cystic fibrosis
A

D. Asthma

This patient has signs and symptoms of asthma. Diagnosis is supported by PEFR variation of at least 20% over 3 days in a week over several weeks or an increase of at least 20% to treatment. Stepwise treatment is based on BTS guidelines.

71
Q
A 26 year old Italian nightclub DJ presents with abdominal pain. On enquiry he has been unwell with a productive cough, fever and breathlessness. On examination his heart rate is 110bpm and his blood pressure is 110/75. His abdomen is soft, non tender.
What is the most likely diagnosis?
A. Emphysema
B. Tuberculosis
C. HIV
D. Asthma
E. Lung abcess
F. Pancreatitis
G. Lung cancer
H. Bronchiectasis
I. Chronic bronchitis
J. Pneumonia
K. Cystic fibrosis
A

J. Pneumonia

This is basal pneumonia which can present with upper abdominal pain. The symptoms this patient gives are consistent with pneumonia. Treatment is guided by the CURB-65 score. A CXR is the most specific and sensitive test available and antibiotics are indicated. CXR may show airspace shadowing with air bronchograms. Make sure you can spot consolidation on a CXR.

72
Q

A 65 year old American woman suffering from chronic bronchitis presents with weight loss, worsening of cough and red blood in sputum. She recently gave up smoking after being a heavy smoker since she was a teenager.

What is the most likely diagnosis?
A. Emphysema
B. Tuberculosis
C. HIV
D. Asthma
E. Lung abcess
F. Pancreatitis
G. Lung cancer
H. Bronchiectasis
I. Chronic bronchitis
J. Pneumonia
K. Cystic fibrosis
A

G. Lung cancer

Weight loss combined with worsening respiratory symptoms and the past history of smoking point to malignancy. First line treatment aims at surgical resection if possible. Small cell lung cancer is treated with chemotherapy and is associated with SIADH and ectopic ACTH. Non-small cell lung cancer is more often associated with clubbing. Squamous cell carcinoma is associated with PTHrp release and is treated with radiotherapy. Adenocarcinomas are usually located peripherally in the lung and are more common in non-smokers although most cases are still associated with smoking.

73
Q

A 3yr old Caucasian child is brought into clinic by his mother who is concerned about his cough. Looking through his notes you find out he has a history of chest infection. However the only medication he is currently taking is a steroid cream for his eczema. You discover that he opens his bowels twice a day and has offensive loose stools. On examination his height is on the 91st centile and his weight is below the 25th centile.

What is the most likely diagnosis?
A. Emphysema
B. Tuberculosis
C. HIV
D. Asthma
E. Lung abcess
F. Pancreatitis
G. Lung cancer
H. Bronchiectasis
I. Chronic bronchitis
J. Pneumonia
K. Cystic fibrosis
A

K. Cystic fibrosis

This infant has presented with cystic fibrosis which is an autosomal recessive condition characterised by a mutation in the CFTR gene on chromosome 7 (delta-F508). In this case there is failure to thrive, recurrent chest infections and evidence of pancreatic insufficiency (steatorrhoea). The most conclusive diagnostic test is the sweat test which is pisitive if sweat chloride is >60mmol/L. Serum IRT from a heel prick blood spot allows screening of newborns. CF is a genetic condition with abnormal salt and water transport due to mutations in the CFTR (an apical anion channel). Heterozygotes generally do not demonstrate disease.

74
Q

A 37 year old Malaysian female presents with a productive cough and purulent sputum and night sweats. What concerns her most is the bright red blood which is often also in the sputum.

What is the most likely diagnosis?
A. Emphysema
B. Tuberculosis
C. HIV
D. Asthma
E. Lung abcess
F. Pancreatitis
G. Lung cancer
H. Bronchiectasis
I. Chronic bronchitis
J. Pneumonia
K. Cystic fibrosis
A

B. Tuberculosis

It is important to have a high level of suspicion when evaluating patients with risk factors who present with suggestive symptoms. Night sweats, fever, malaise, cough, haemoptysis and erythema nodosum are all suggestive. Other key risk factors for pulmonary TB include exposure to infection and returning from or being born in a high-risk region such as Asia, Africa and Latin America. If TB is suspected, the patient should be placed in isolation and a CXR obtained with 3 sputum samples cultured for AFB being the gold standard of diagnosis. Culture takes several weeks so sputum smears will be done before culture results are known. Interferon-gamma release assays (IGRAs) are now used by some hospitals to rapidly determine a patient’s TB status. All patients who have TB should be tested for HIV within 2 months of diagnosis.

75
Q

A 35 year old man presents to A&E with a short history of haemoptysis & breathlessness. His pulse is 125bpm & he has recently travelled to Australia. Chest examination is unremarkable. He is allergic to contrast agents.

Which single investigation is the most appropriate to confirm the likely diagnosis?
A. Lung function tests
B. History only
C. CTPA
D. Sputum cultures
E. ABG
F. CT head
G. V/Q scan
H. Chest x-ray
I. Clotting screen
J. MRA
K. Bronchoscopy
L. D-dimer
A

G. V/Q scan

This patient has a PE. The study of choice is a CTPA with direct visualisation of the thrombus. If there is a contraindication to a CT scan such as contrast allergy (in this case) or pregnancy, then a V/Q scan is indicated. If a V/Q scan is not possible, alternatives such as MRA can be requested. It is worth noting that in patients with cardiopulmonary disease, these tests may not be accurate. A TTE can also be used to detect RV strain seen with PE. Patients with a high clinical suspicion of PE should be anticoagulated while waiting a definitive diagnosis unless contraindicated. The underlying pathophysiology is based on Virchow’s triad. SOB and chest pain are common symptoms and there may also be haemoptysis. Strong risk factors include DVT, obesity, surgery in the past 2 months, prolonged bed rest, malignancy (which this patient has), previous VTE, pregnancy and the thrombophilias such as factor V Leiden.

76
Q

A 30 year old Indian lady who has recently returned from a 4 month trip to Delhi complains of a cough & haemoptysis & night sweats over the last month.

Which single investigation is the most appropriate to confirm the likely diagnosis?
A. Lung function tests
B. History only
C. CTPA
D. Sputum cultures
E. ABG
F. CT head
G. V/Q scan
H. Chest x-ray
I. Clotting screen
J. MRA
K. Bronchoscopy
L. D-dimer
A

D. Sputum cultures

This sounds like TB. The patient should be placed in isolation and a CXR obtained with 3 sputum samples cultured for AFB being the gold standard of diagnosis. Treatment is with anti-TB medication: generally 2 months of rifampicin, isoniazid (given with pyridoxine to prevent associated neuropathy), pyrazinamide and ethambutol followed by 4 months of rifampicin and isoniazid alone. Treatment of MDR TB is more complicated and involves longer treatment with more drugs. Note that CNS TB requires 12 months of treatment.

77
Q

A 19 year old man who has been intubated due to a recent RTA is recovering well & is extubated. He complains of coughing up a small amount of blood streaked phlegm.

Which single investigation is the most appropriate to confirm the likely diagnosis?
A. Lung function tests
B. History only
C. CTPA
D. Sputum cultures
E. ABG
F. CT head
G. V/Q scan
H. Chest x-ray
I. Clotting screen
J. MRA
K. Bronchoscopy
L. D-dimer
A

B. History only

This is a result of intubation which has caused some iatrogenic trauma to this patient’s upper airway.

78
Q

A 55 year old lawyer who has had a chronic cough for 3 months complains of 1 episode of haemoptysis. She is a heavy smoker (about 40/day) & has experienced some recent weight loss.

Which single investigation is the most appropriate to confirm the likely diagnosis?
A. Lung function tests
B. History only
C. CTPA
D. Sputum cultures
E. ABG
F. CT head
G. V/Q scan
H. Chest x-ray
I. Clotting screen
J. MRA
K. Bronchoscopy
L. D-dimer
A

K. Bronchoscopy

This sounds like bronchial carcinoma from the history of smoking, respiratory complaints and weight loss. Initial investigation of bronchial carcinoma is with a CXR but definitive diagnosis relies on pathological confirmation from a tissue sample, often obtained from bronchoscopy. During bronchoscopy, endobronchial masses can be biopsied and washings/alveolar lavage can also be performed for cytological analysis. Trans-thoracic needle aspiration may be needed for peripheral lesions that cannot be reached by bronchoscopy. First line treatment aims at surgical resection if possible.

79
Q

A 40 year old man complains of breathlessness & coughing up blood for 3 days. He mentions that prior to this he had fallen onto the corner of a table. Breath sounds seem slightly reduced on the right had side.

Which single investigation is the most appropriate to confirm the likely diagnosis?
A. Lung function tests
B. History only
C. CTPA
D. Sputum cultures
E. ABG
F. CT head
G. V/Q scan
H. Chest x-ray
I. Clotting screen
J. MRA
K. Bronchoscopy
L. D-dimer
A

H. Chest x-ray

This is chest trauma which has obviously caused some damage. This will need to be visualised by performing a chest XR. A CT scan may also be necessary. There is a chance of atelectasis given this patient’s history and findings on examination.

80
Q

A young 23 year old allergic atopic asthmatic woman becomes acutely breathless. She has just taken an Aspirin for headache. Her neck is swollen, eyes puffy, & her breathing is noisy, with marked wheeze throughout the lung fields.

What is the most appropriate treatment?
A. Intramuscular adrenaline
B. Beta agonist via metered dose inhaler
C. Influenza immunisation
D. 40% continuous oxygen
E. Amoxycillin intravenously
F. Long acting beta agonist
G. Intravenous aminophylline
H. 24% continuous oxygen
I. Antibiotic reserve
J. Intravenous hydrocortisone
K. Oral prednisolone
L. Beta agonist via nebuliser
A

A. Intramuscular adrenaline

There is a sudden onset of respiratory (and cardiovascular) complaints with the recently given aspirin tablet. This patient is having an anaphylactic reaction and the airway needs to be promptly secured and prompt treatment started with adrenaline. IM adrenaline must not be delayed and the anterolateral thigh is the preferred location, with repeated doses as necessary every 10 minutes or so. A 1:1000 solution is used of 0.3-0.5mg adrenaline. Fluid replacement with IV saline is also indicated to correct the intravascular volume redistribution.

81
Q

A 70 year old man has been newly diagnosed with stage I COPD and has been given inhaled salbutamol PRN and has agreed to stop smoking. What additional measure is needed?

What is the most appropriate treatment?
A. Intramuscular adrenaline
B. Beta agonist via metered dose inhaler
C. Influenza immunisation
D. 40% continuous oxygen
E. Amoxycillin intravenously
F. Long acting beta agonist
G. Intravenous aminophylline
H. 24% continuous oxygen
I. Antibiotic reserve
J. Intravenous hydrocortisone
K. Oral prednisolone
L. Beta agonist via nebuliser
A

C. Influenza immunisation

The GOLD guidelines published in April 2010 provide a framework for a stepwise approach to treating COPD. Stages of COPD are based on predicted FEV1. For all stages of COPD, influenza vaccination is given yearly and pneumococcal vaccine is given every 5 years. Long term oxygen is only added if there is chronic respiratory failure in stage IV disease. Stage II involves the addition of long acting bronchodilators and rehabilitation. Stage III involves adding inhaled GCs if there are repeated exacerbations.

82
Q

A 64 year old man has become acutely breathless over the last 4 days, with a productive cough, green sputum & chest tightness. He is a longstanding smoker. Clinically he is hypoxic, with tachycardia, tachypnoea, & central cyanosis. His ABG are as follows: pH 7.35, PaO2 6.7kPa, PaCO2 7.8kPa. He has been given nebulised bronchodilators & intravenous antibiotics & steroids.

What is the most appropriate treatment?
A. Intramuscular adrenaline
B. Beta agonist via metered dose inhaler
C. Influenza immunisation
D. 40% continuous oxygen
E. Amoxycillin intravenously
F. Long acting beta agonist
G. Intravenous aminophylline
H. 24% continuous oxygen
I. Antibiotic reserve
J. Intravenous hydrocortisone
K. Oral prednisolone
L. Beta agonist via nebuliser
A

H. 24% continuous oxygen

This patient with COPD is having an acute exacerbation. He is being treated accordingly but will need supplemental oxygen as a result of his ABG results. >8kPa is an acceptable level of arterial oxygenation or SaO2 >90%. Ceftriaxone is an acceptable antibiotic. High risk individuals should get tazocin or meropenem (especially if pseudomonas is suspected). Check local antibiotic prescribing policies. Chronic oxygen therapy is indicated for patients with PaO2 <7.3kPa or with evidence of pulmonary hypertension, pulmonary oedema or polycythaemia if the PaO2 falls between 7.3 and 8kPa.

83
Q

A 9 year old girl attends her GP surgery as an emergency with wheezing, difficulty in breathing & cough. Her PEFR is 250l/min, expected PEFR 500. There is no sign of infection on her chest examination. She is admitted to hospital and given inhaled salbutamol although shows an incomplete response. Which treatment should be given next?

What is the most appropriate treatment?
A. Intramuscular adrenaline
B. Beta agonist via metered dose inhaler
C. Influenza immunisation
D. 40% continuous oxygen
E. Amoxycillin intravenously
F. Long acting beta agonist
G. Intravenous aminophylline
H. 24% continuous oxygen
I. Antibiotic reserve
J. Intravenous hydrocortisone
K. Oral prednisolone
L. Beta agonist via nebuliser
A

K. Oral prednisolone

Acute asthma exacerbation this time. Bear in mind that treatment guidelines differ for adults and children. The PEFR is 50% predicted making this a moderate exacerbation of asthma (40-69%). A mild exacerbation is defined by a PEFR of 70% of more of predicted whereas severe is defined as 26 to 39% with life-threatening falling under 25%. Note that these values are for children. For moderate exacerbations like this, an oral corticosteroid needs to be prescribed alongside inhaled SABA. If this were life-threatening or severe, then IV may be indicated. It is worth knowing the symptoms and signs of mild, moderate and severe/life threatening asthma. The treatment guidelines that you need to familiarise yourself with tend to be for adults.

84
Q

A 70 year old male smoker with a 5 year history of productive cough presents with breathlessness. On examination you find hyperinflation & diminished breath sounds when examining the chest.

What is the most likely diagnosis?
A. Anaemia
B. COPD
C. Pulmonary embolus
D. Anxiety
E. Left ventricular failure
F. Epiglottitis
G. Mitral stenosis
H.Thyrotoxicosis
I. Mitral regurgitation
J. Asthma
K. Pneumothorax
L. Aspirin poisoning
M. Pneumonia
A

B. COPD

Smoking is the most important risk factor, accounting for 90% of COPD. COPD has an insidious onset and usually presents in older people with a history of cough, wheeze and SOB. Patients with COPD are at a higher risk of infections and are vaccinated against influenza annually and pneumococcal pneumonia every 5 years. Spirometry is the gold standard for diagnosis, with FEV1/FVC ratio <70% with no evidence of reversibility (unlike asthma) being indicative. These examination findings are consistent with emphysematous hyperexpanded lungs consistent with COPD.

85
Q

A 40 year old widowed female presents with a 3 week history of shortness of breath. She also complains of chronic fatigue. On examination she is pale with a pulse of 120.

What is the most likely diagnosis?
A. Anaemia
B. COPD
C. Pulmonary embolus
D. Anxiety
E. Left ventricular failure
F. Epiglottitis
G. Mitral stenosis
H.Thyrotoxicosis
I. Mitral regurgitation
J. Asthma
K. Pneumothorax
L. Aspirin poisoning
M. Pneumonia
A

A. Anaemia

Anaemia is defined by haemoglobin concentration (<12 females and <14 males or there abouts – different books quote slightly different figures for normal). The most common cause and a likely cause in this patient is iron deficiency and females are affected more so than males. Anaemias are classified into microcytic, normocytic and macrocytic by MCV which guides the investigation of the cause of anaemia. Pallor is a finding on examination. There may be koilonychia, angular stomatitis and glossitis. SOB is a non-specific complaint which is not particularly common though patients often complain of fatigue (like this patient) and weird cravings. Dysphagia may also be a presentation of IDA with swallowing difficulties at the level of the cricoid consistent with Plummer-Vinson syndrome – an uncommon presentation but somewhat seen in EMQs. This patient’s anaemia will need to be investigated as the cause is unclear.

86
Q

An 85 year old male presents with shortness of breath associated with confusion. On examination there is decreased expansion on the left side & the patient with respiratory rate of 35/min.

What is the most likely diagnosis?
A. Anaemia
B. COPD
C. Pulmonary embolus
D. Anxiety
E. Left ventricular failure
F. Epiglottitis
G. Mitral stenosis
H.Thyrotoxicosis
I. Mitral regurgitation
J. Asthma
K. Pneumothorax
L. Aspirin poisoning
M. Pneumonia
A

M. Pneumonia

87
Q

A 75 year old recently widowed male smoker with a history of angina presents with shortness of breath. He has also vomited & complains of a ringing in his ears. On examination the patient has a BP of 80/50mmHg & fine crackles at both lung bases.

What is the most likely diagnosis?
A. Anaemia
B. COPD
C. Pulmonary embolus
D. Anxiety
E. Left ventricular failure
F. Epiglottitis
G. Mitral stenosis
H.Thyrotoxicosis
I. Mitral regurgitation
J. Asthma
K. Pneumothorax
L. Aspirin poisoning
M. Pneumonia
A

L. Aspirin poisoning

This patient has angina so probably has a stash of aspirin. Tinnitis is common in the early stages of acute salicylate poisoning and reflects CNS toxicity. There may also be deafness and both are reversible. GIT decontamination should be considered as an adjunct on arrival to A&E and activated charcoal can be given. The mainstay of treatment is alkaline diuresis induced by an infusion of sodium bicarbonate. In cases of severe poisoning, it is still started as a bridge to haemodialysis.

88
Q

An 80 year old female with a history of rheumatic fever in childhood & palpitations presents with shortness of breath. On examination he has an irregularly irregular pulse of 120bpm & loud first heart sound.

What is the most likely diagnosis?
A. Anaemia
B. COPD
C. Pulmonary embolus
D. Anxiety
E. Left ventricular failure
F. Epiglottitis
G. Mitral stenosis
H.Thyrotoxicosis
I. Mitral regurgitation
J. Asthma
K. Pneumothorax
L. Aspirin poisoning
M. Pneumonia
A

G. Mitral stenosis

Practically every single case of mitral stenosis is caused by rheumatic heart disease. The major criteria for rheumatic fever can be remember by CASES: carditis, arthritis, Sydenham’s chorea, erythema marginatum and subcutaneous nodules. The process tends to also cause regurgitation. Mitral stenosis is characteristically a grade 1-2 low pitch murmur heard in mid-diastole which has a rumbling nature and there is no radiation. There can be an associated malar flush, tapping apex beat and a diastolic thrill palpable at the apex, in the 5th intercostal space in the MCL. The first heart sound is also characteristically loud and often this is the most striking feature on ascultation. It is a difficult murmur to pick up so if you are ever asked at this stage to spot this murmur, it will most likely be based on the loud S1. Mitral stenosis is associated with AF caused by LA enlargement (seen by the irregularly irregular pulse).